3) Cardinal Numbers - Suggested Solutions to Jech's Set Theory

$ \newcommand{\func}{\operatorname} \newcommand{\lcm}{\func{lcm}} \newcommand{\hl}[1]{\textbf{#1}} \newcommand{\lf}{\lfloor} \newcommand{\rf}{\rfloor} \newcommand{\f}[1]{\left\lf #1 \right\rf} \newcommand{\bb}[1]{\left( #1 \right)} \newcommand{\Sb}[1]{\left\lbrack #1 \right\rbrack} \renewcommand{\ss}[1]{\left\lbrace #1 \right\rbrace} \renewcommand{\mod}[1]{\left\lvert #1 \rightrvert} \newcommand{\R}{\boldsymbol{R}} \newcommand{\C}{\boldsymbol{C}} \newcommand{\Z}{\boldsymbol{Z}} \newcommand{\Q}{\boldsymbol{Q}} \renewcommand{\P}{\mathbb{P}} \newcommand{\Po}{\mathcal{P}} \newcommand{\F}{\mathcal{F}} \newcommand{\Zz}{\Z_{\geq 0}} \renewcommand{\c}[1]{\left\langle #1 \right\rangle} \renewcommand{\mod}[1]{\left\lvert #1 \right\rvert} \newcommand{\norm}[1]{\left\lVert #1 \right\rVert} \renewcommand{\bar}[1]{\overline{#1}} \newcommand{\eqbreak}{\phantom{{} = {}}} \newcommand{\impliesbreak}{\phantom{{} \implies {}}} \newcommand{\iffbreak}{\phantom{{} \iff {}}} \newcommand{\del}{\partial} \renewcommand{\d}[1]{\, \mathrm{d} #1} \newcommand{\dydt}{\frac{\d{y}}{\d{t}}} \newcommand{\dydx}{\frac{\d{y}}{\d{x}}} \newcommand{\pd}[2]{\frac{\del #1}{\del #2}} \newcommand{\pp}[3]{\frac{\del^2 #1}{\del #2 \del #3}} \newcommand{\dd}[2]{\frac{\mathrm{d} #1}{\mathrm{d} #2}} \newcommand{\Inn}{\func{Inn}} \newcommand{\Aut}{\func{Aut}} \newcommand{\diag}{\func{diag}} \newcommand{\id}{\func{id}} \newcommand{\GL}{\func{GL}} \newcommand{\SL}{\func{SL}} \newcommand{\End}{\func{End}} \newcommand{\Hom}{\func{Hom}} \newcommand{\Tr}{\func{Tr}} \newcommand{\rank}{\func{rank}} \newcommand{\ORD}{\mathbf{ORD}} \newcommand{\OD}{\mathbf{OD}} \newcommand{\HOD}{\mathbf{HOD}} \newcommand{\ZF}{\mathsf{ZF}} \newcommand{\ZFC}{\mathsf{ZFC}} \newcommand{\ZC}{\mathsf{ZC}} \renewcommand{\AC}{\mathsf{AC}} \newcommand{\DC}{\mathsf{DC}} \newcommand{\CC}{\mathsf{CC}} \newcommand{\CH}{\mathsf{CH}} \newcommand{\SH}{\mathsf{SH}} \newcommand{\KH}{\mathsf{KH}} \newcommand{\GCH}{\mathsf{GCH}} \newcommand{\PA}{\mathsf{PA}} \newcommand{\BST}{\mathsf{BST}} \newcommand{\MA}{\mathsf{MA}} \newcommand{\Con}{\func{Con}} \newcommand{\ON}{\mathbf{ON}} \newcommand{\dom}{\func{dom}} \newcommand{\ran}{\func{ran}} \newcommand{\pred}{\func{pred}} \newcommand{\mos}{\func{mos}} \newcommand{\WF}{\mathbf{WF}} \newcommand{\type}{\func{type}} \newcommand{\V}{\mathbf{V}} \renewcommand{\L}{\mathcal{L}} \newcommand{\cl}{\func{cl}} \newcommand{\trcl}{\func{tc}} \newcommand{\TC}{\func{TC}} \newcommand{\A}{\mathfrak{A}} \newcommand{\B}{\mathfrak{B}} \newcommand{\add}{\func{add}} \newcommand{\cov}{\func{cov}} \newcommand{\non}{\func{non}} \newcommand{\cf}{\func{cf}} \newcommand{\forces}{\Vdash} \newcommand{\one}{\mathbbm{1}} \newcommand{\restrictedto}{\mathord{\upharpoonright}} \newcommand{\Fn}{\func{Fn}} \newcommand{\Col}{\func{Col}} \newcommand{\bigtriangle}{\triangle} \newcommand{\U}{\mathcal{U}} \newcommand{\W}{\mathcal{W}} \newcommand{\N}{\mathcal{N}} \newcommand{\Seq}{\mathit{Seq}} \newcommand{\lh}{\func{lh}} \newcommand{\Ult}{\func{Ult}} \newcommand{\In}{\func{In}} \newcommand{\otp}{\func{otp}} \newcommand{\symdiff}{\, \triangle \,} \newcommand{\crit}{\func{crit}} \newcommand{\sat}{\func{sat}} \renewcommand{\int}{\func{int}} \newcommand{\length}{\func{length}} \newcommand{\Lim}{\func{Lim}} \newcommand{\Succ}{\func{Succ}} \newcommand{\G}{\mathcal{G}} \newcommand{\T}{\mathcal{T}} \newcommand{\ext}{\func{ext}} \newcommand{\height}{\func{height}} \renewcommand{\diamond}{\diamondsuit} \newcommand{\D}{\mathcal{D}} \newcommand{\NS}{\mathrm{NS}} $


Exercise 3.1.


Exercise 3.1(i).

Solution. Let $X$ be a set with finite cardinality $\vert X\vert = n$. We induct on $n$. If $n = 0$, then the only subset is $\emptyset$, which is finite. If $\vert X\vert = n + 1$, then let $Y \subseteq X$. We consider two cases.
  1. If $Y = X$, then since $X$ is finite, so is $Y$.
  2. If $Y \subsetneq X$, then let $x_0 \in X - Y$. Then $Y \subseteq X - \lbrace x_0\rbrace$. By Lemma 1.12.A, $\vert X - \lbrace x_0\rbrace\vert = n$, so by induction hypothesis $Y$ is also finite.

$\square$




Exercise 3.1(ii).

Solution. First assume $A \cap B = \emptyset$. Suppose $\vert A\vert = m$ and $\vert B\vert = n$. First suppose $A \cap B = \emptyset$. Let $f : A \to m$ and $g : B \to n$ be bijections. Then define $h : A \sqcup B \to m + n$ by:
$$ \begin{align*}
h(x) :=
\begin{cases}
f(x), &\text{if $x \in A$} \\
g(x) + n, &\text{if $x \in B$}
\end{cases}
\end{align*}$$ This is well-defined as $A \cap B = \emptyset$. It's easy to see that $h$ is a bijection.

For the general case, we observe that $A \cup B = A \cup (B - A)$, $A \cap (B - A) = \emptyset$, and $B - A \subseteq B$ so it is also finite by Exercise 3.1(i).

$\square$




Exercise 3.1(iii).

Solution. This follows from Lemma 3.3, which asserts that if $\vert X\vert = n$ then $\vert P(X)\vert = 2^n$, which is finite.

$\square$




Exercise 3.1(iv).

Solution. Let $X$ be a finite set and let $f : X \to Y$ be a mapping to some (not necessarily finite) set $Y$. Then, considering $f : X \to f"(X)$ (by adjusting the codomain), we obtain a bijection between $X$ and $f"(X)$. If $g : X \to n$ is a bijection, then $g \circ f^{-1} : f"(X) \to n$ is also a bijection, so $f"(X)$ is finite.

$\square$




Exercise 3.2.


Exercise 3.2(i).

Solution. Note that a set is at most countable iff $\vert X\vert \leq \vert \omega\vert $, iff there exists an injective mapping $X \to \omega$.

Let $X$ be a countable set, and let $Y \subseteq X$. Consider the inclusion map $i : Y \to X$, which is of course injective. Since $X$ is countably, there exists a bijective map $f : X \to \omega$. Then $f \circ i : Y \to \omega$ is an injective mapping, so $Y$ is at most countable.

$\square$




Exercise 3.2(ii).

Solution. Suppose $X_1,\dots,X_n$ are countable sets. Let $X := \bigcup_{i=1}^n X_i$. Since $X_1 \subseteq X$, $\vert X_1\vert \leq \vert X\vert $ so $\vert X\vert \geq \aleph_0$. To finish the proof, we need to find an injective mapping $X \to \omega$, so $\vert X\vert \leq \aleph_0$.

For each $i$, let $f_i : X_i \to \boldsymbol{N}$ be a bijection. Define $f : X \to \omega$ by stipulating that:
$$ \begin{align*}
f(x) = p_i^{f_i(x)}, \;\;\text{if $x \in X_i - \bigcup_{j < i} X_j$}
\end{align*}$$ where $p_i$ denotes the $i^\text{th}$ smallest prime number. It's easy to see that, by fundamental theorem of arithmetic, $f$ is injective.

$\square$




Exercise 3.2(iii).

Solution. Let $X,Y$ be sets, $X$ countable, and let $f : X \to Y$ be a mapping. Let $g : \omega \to X$ be a bijection. Then $f \circ g : \omega \to Y$. Now define $h : Y \to \omega$ by stipulating that:
$$ \begin{align*}
h(y) := \min\lbrace n \in \omega : (g \circ f)(n) = y\rbrace
\end{align*}$$ Then $h$ is well-defined as $\omega$ is well-ordered, and clearly $h$ is injective. Thus there exists an injective map from $Y$ to $\omega$, so it is at most countable.

$\square$




Exercise 3.3.

Solution. We shall show that the $f : \boldsymbol{N} \times \boldsymbol{N} \to \boldsymbol{N}$ in the hint is bijective.

Injective: Suppose $f(m_1,n_1) = f(m_2,n_2)$, so $2^{m_1}(2n_1 + 1) - 1 = 2^{m_2}(2n_2 + 1) - 1 \implies 2^{m_1}(2n_1 + 1) = 2^{m_2}(2n_2 + 1)$. Now the $2^{n_i}$'s are odd, so $\nu_2(2^{m_i}(2n_i + 1)) = m_i$. By Fundamental Theorem of Arithmetic, we have that $m_1 = m_2$. This implies that $2n_1 + 1 = 2n_2 + 1$, which easily gives $n_1 = n_2$. Thus $(m_1,n_1) = (m_2,n_2)$.

Surjective: For any $N \in \boldsymbol{N}$, $N + 1$ is a positive integer, and by the Fundamental Theorem of Arithmetic we may write $N + 1 = 2^mN_2$ for some $m \in \boldsymbol{N}$ and $N_2 \in \boldsymbol{N}$, where $N_2$ is an odd integer. Then $N_2 = 2n + 1$ for some $n \in \boldsymbol{N}$, so $f(m,n) = 2^m(2n + 1) - 1 = N$.

$\square$




Exercise 3.4.


Exercise 3.4(i).

Solution. Let $\boldsymbol{N}^{<\omega}$ denote the set of finite sequences in $\boldsymbol{N}$. Define a map $f : \boldsymbol{N}^{<\omega} \to \boldsymbol{N}$ as follows: For each $s = \c{s(0),\dots,s(n-1)} \in \boldsymbol{N}^{<\omega}$, let:
$$ \begin{align*}
f(s) = p_1^{s(0)+1}\cdots p_n^{s(n-1)+1}
\end{align*}$$ where $p_k$ is the $k^\text{th}$ smallest prime number. The injectivity of this map follows from the Fundamental Theorem of Arithmetic, so $\vert \boldsymbol{N}^{<\omega}\vert \leq \vert \boldsymbol{N}\vert $. On the other hand, the map $n \mapsto \c{n}$ is clearly an injective map $\boldsymbol{N} \to \boldsymbol{N}^{<\omega}$, so $\vert \boldsymbol{N}^{<\omega}\vert \geq \vert \boldsymbol{N}\vert $. By Cantor-Bernstein Theorem (Theorem 3.2), we have that $\vert \boldsymbol{N}^{<\omega}\vert = \vert \boldsymbol{N}\vert $, so $\boldsymbol{N}^{<\omega}$ is countably infinite.

$\square$




Exercise 3.4(ii).

Solution. WLOG we may show the statement for finite subsets of $\boldsymbol{N}$, the general case follows from establishing a bijection between $\boldsymbol{N}$ and the arbitrary countable set.

Clearly $\vert \boldsymbol{N}\vert \leq \vert [\boldsymbol{N}]^{<\omega}\vert $ by the injective map $n \mapsto \lbrace n\rbrace$. On the other hand, for each $\lbrace a_0,\dots,a_{n-1}\rbrace \in [\boldsymbol{N}]^{<\omega}$, where $a_0 < \cdots < a_{n-1}$, define the map from $[\boldsymbol{N}]^{<\omega}$ to $\boldsymbol{N}^{<\omega}$ by:
$$ \begin{align*}
\lbrace a_0,\dots,a_{n-1}\rbrace \mapsto \c{a_0,\dots,a_{n-1}}
\end{align*}$$ Clearly this map is injective, so $\vert [\boldsymbol{N}]^{<\omega}\vert \leq \vert \boldsymbol{N}^{<\omega}\vert $. By Exercise Exercise 3.4(i), we have that $\vert [\boldsymbol{N}]^{<\omega}\vert = \vert \boldsymbol{N}\vert $, so $[\boldsymbol{N}]^{<\omega}$ is countable.

$\square$




Exercise 3.5.


Lemma 3.5.A. For all ordinals $\alpha$, we have $\alpha \leq \omega^\alpha$.

Proof. We induct on $\alpha$. For $\alpha = 0$ we have $\omega^0 = 1$. If $\beta \leq \omega^\beta$, then:
$$ \begin{align*}
\beta + 1 \leq \omega^\beta + 1 \leq \omega^\beta + \omega^\beta = \omega^\beta \cdot 2 \leq \omega^\beta \cdot \omega = \omega^{\beta+1}
\end{align*}$$ If $\alpha$ is a limit ordinal, then:
$$ \begin{align*}
\alpha = \lim_{\beta \to \alpha} \beta \leq \lim_{\beta \to \alpha} \omega^\beta = \omega^\alpha
\end{align*}$$

$\blacksquare$



Solution. For the first part, we induct on $\alpha$. For $\alpha = 0$, clearly $\Gamma(0 \times 0) = 0 \leq 1 = \omega^0$. If $\alpha > 0$ is a limit ordinal, then by continuity we have:
$$ \begin{align*}
\Gamma(\alpha \times \alpha) = \bigcup_{\beta<\alpha} \Gamma(\beta \times \beta) \leq \bigcup_{\beta<\alpha} \omega^\beta = \omega^\alpha
\end{align*}$$ Now suppose $\alpha = \beta + 1$. Since $(\gamma,\delta) \in \beta \times \beta$ iff $(\gamma,\delta) < (0,\beta)$ (so $\Gamma(0,\beta) \leq \omega^\beta$), we have that:
$$ \begin{align*}
\Gamma(\alpha \times \alpha) - \Gamma(\beta \times \beta) = \underbrace{\lbrace (\gamma,\beta) : \gamma < \beta\rbrace}_{\text{order type $\beta$}} \cup \underbrace{\lbrace (\beta,\gamma) : \gamma \leq \beta\rbrace}_{\text{order type $\beta + 1$}}
\end{align*}$$ Thus, this is of order type $\beta\cdot 2 + 1$, so by Lemma 3.5.A:
$$ \begin{align*}
\Gamma(\alpha \times \alpha) &= \Gamma(\beta + \beta) + \beta \cdot 2 + 1 \\
&\leq \omega^\beta + \beta + \beta + 1 \\
&\leq \omega^\beta + \omega^\beta + \omega^\beta + \omega^\beta \\
&= \omega^\beta \cdot 4 \\
&\leq \omega^\beta \cdot \omega \\
&= \omega^{\beta+1}
\end{align*}$$ The second part of the exercise is false. However, it is true that the fixed points of $\gamma(\alpha) = \Gamma(\alpha \times \alpha)$ are precisely the multiplicatively indecomposable ordinals. See Exercise I.11.7 of Kunen's Set Theory. The notion of indecomposable ordinals introduced in Exercise 2.13 is also called additively indecomposable.

We shall show that $\Gamma(\omega^2 \times \omega^2) > \omega^2$. To do this, it suffices to show that $\Gamma((\omega + \omega) \times (\omega + \omega)) \geq \omega^2$. For each $n \in \omega$, the set:
$$ \begin{align*}
\lbrace (\alpha, \omega + n) : \alpha < \omega + n\rbrace \cup \lbrace (\omega + n,\alpha) : \alpha \leq \omega + n\rbrace
\end{align*}$$ is of order type $(\omega + n) + (\omega + n + 1) = \omega \cdot 2 + n + 1$. Thus, it's easy to prove inductively that for each $n \in \omega$, we have $\Gamma((\omega + n) \times (\omega + n)) \geq \omega \cdot (2n + 1)$. Therefore:
$$ \begin{align*}
\Gamma((\omega + \omega) \times (\omega + \omega)) \geq \lim_{n \in \omega} \omega \cdot (2n + 1) = \omega^2
\end{align*}$$

$\square$




Exercise 3.6.


Lemma 3.6.A. Let $\alpha < \omega_\beta$ be an ordinal. Let $\alpha^{<\omega}$ be the set of all finite sequences of ordinals below $\alpha$. Then $\vert \alpha^{<\omega}\vert < \kappa$.

Proof. Let $f : \alpha \to \vert \alpha\vert $ be a bijection. By Theorem 3.5 we know that $\Gamma(\vert \alpha\vert \times \vert \alpha\vert ) = \vert \alpha\vert $. Thus, we obtain a choiceless bijection $g := f^{-1} \circ \Gamma \circ (f,f) : \alpha \times \alpha \to \alpha$. Let $g_2 := g$, and inductively define $g_n : \alpha^n \to \alpha$ by $g_n := (g_{n-1},g)$. With this, we may define $h : \alpha^{<\omega} \to \alpha \times \omega$ by:
$$ \begin{align*}
h(s) := (g_{\length(s)}(s),\length(s))
\end{align*}$$ Then clearly $h$ is one-to-one, so we have:
$$ \begin{align*}
\vert \alpha^{<\omega}\vert < \vert \alpha \times \vert \omega\vert = \vert \alpha\vert \cdot \aleph_0 \leq \vert \alpha\vert < \omega_\beta
\end{align*}$$

$\blacksquare$



Solution. We require the Axiom of Choice in this solution.

We first define a function $\height$ on finite sequences of ordinals, in which if $f$ is such a sequence then:
$$ \begin{align*}
\height(s) := \max\lbrace \length(s),\max\lbrace s(i) : i < \length(s)\rbrace\rbrace
\end{align*}$$ We shall define an ordering as follows: Given two finite sequences of ordinals $s$ and $t$, we say that $s < t$ iff one of the following holds:
  1. $\height(s) < \height(t)$.
  2. $\height(s) = \height(t)$ and $\length(s) < \length(t)$.
  3. $\height(s) = \height(t)$, $\length(s) = \length(t)$ and there exists some $k < \length(s)$ such that $s(k) < t(k)$, and $s(i) = t(i)$ for all $i < k$.
As an example, an initial segment of this ordering is as follows:
$$ \begin{gather*}
\emptyset < (0) < (1) < (2) < (0,0) < (0,1) < (0,2) < (1,0) < (1,1) < (1,2) \\
< (2,0) < (2,1) < (2,2) < (3) < (0,3) < (1,3) < (2,3) < (3,3) < (0,0,0) < \dots
\end{gather*}$$ One can check that $<$ is a linear order (transitivity is the most tedious to check, but it can be done by considering cases). We first show that $<$ is a well-order. Let $X \subseteq \boldsymbol{\mathrm{ORD}}^{<\omega}$. Let $Y \subseteq X$ be the non-empty subset of all elements of $X$ of minimal height, then let $Z \subseteq Y$ be the non-empty subset of all elements of $Y$ of minimal length. Thus, all elements in $Z$ have equal height and length, and $(\forall z \in Z)(\forall x \in X - Z) \, z < x$. Thus, it suffices to show that $Z$ has a $<$-minimal element.

Suppose all elements of $Z$ has length $n$. For each $i < n$, let:
  1. $Z_0 := \lbrace z \in Z : z(0) = \min\lbrace z'(0) : z' \in Z\rbrace\rbrace$.
  2. $Z_i := \lbrace z \in Z_{i-1} : z(i) = \min\lbrace z'(i) : z' \in Z\rbrace\rbrace$.
By well-ordering of ordinals all of $Z_i$'s are non-empty. Let $z \in Z_{n-1}$. Then $z$ is the minimal element in $Z$ - otherwise, if $z' < z$, then since $\height(z) = \height(z')$ and $\length(z) = \length(z')$, we have that $z'(k) < z(k)$ for some $k < n$, and $z'(i) = z(i)$ for $i < k$. Since $z(i)$ is minimal for all $i$, we have that $z'(i)$ is minimal for all $i < k$, therefore $z' \in Z_{k-1}$. But $z \in Z_k$ and $z'(k) < z(k)$, contradicting minimality of $z(k)$.

Thus, we may denote $\Lambda(s)$ as the order type of the set of all finite sequences below $s$ under $<$. Let $\Lambda(\alpha^{<\omega}) := \Lambda((\alpha))$. $\Lambda(\omega_\alpha^{<\omega}) = \omega_\alpha$ for all $\alpha$. We first observe that $(\beta) < (\omega_\alpha)$ for all $\beta < \omega_\alpha$, so $\Lambda(\omega_\alpha^{<\omega}) \geq \omega_\alpha$. Now suppose for a contradiction that $\Lambda(\omega_\alpha^{<\omega}) > \omega_\alpha$. Let $s$ be the $<$-minimal sequence such that $\Lambda(s) = \omega_\alpha$. Let $\delta < \omega_\alpha$ such that $\delta > s(i)$ for all $i < \length(s)$. Then $\omega_\alpha \subseteq \Lambda(\delta^{<\omega})$. Now observe that $\Lambda(\delta^{<\omega}) \subseteq \delta^{<\omega}$, so $\vert \delta^{<\omega}\vert \geq \aleph_\alpha$. This contradicts Lemma 3.6.A.

$\square$




Exercise 3.7.

Solution. Let $f : \omega_\alpha \to B$ be an onto function. Define $g : B \to \omega_\alpha$ by stipulating that for $x \in B$, $g(x) = \min\lbrace f_{-1}(\lbrace x\rbrace)\rbrace$. This is well defined as for $x \neq y$, $f_{-1}(\lbrace x\rbrace)$ and $f_{-1}(\lbrace y\rbrace)$ are disjoint. They are also non-empty as $f$ is onto. Then $g$ is a one-to-one function on $B$ into $\omega_\alpha$, so $\vert B\vert \leq \vert \omega_\alpha\vert = \aleph_\alpha$.

$\square$




Exercise 3.8.

Solution. We note that by restricting the well-order in Exercise 3.6 to finite sequences of $\omega_\alpha$, we obtain a choiceless well-order in $\omega_\alpha^{<\omega}$, along with the order type $\Lambda$. Then $\Lambda^{-1}(\omega_\alpha) = \omega_\alpha^{<\omega}$, so $\omega_\alpha^{<\omega}$ is a projection of $\omega_\alpha$. Clearly the set of all finite subsets of $\omega_\alpha$ is a projection of $\omega_\alpha^{<\omega}$, so the set of all finite subsets of $\omega_\alpha$ is a projection of $\omega_\alpha$ itself. By Exercise 3.7, this set has cardinality $\leq \aleph_\alpha$. On the other hand, the map $\beta \mapsto \lbrace \beta\rbrace$ is a one-to-one function on $\omega_\alpha$ to this set, so it has cardinality $\geq \aleph_\alpha$. Thus equality follows.

$\square$




Exercise 3.9.

Solution. Consider the function $g$ in the hint. It suffices to show that the function $g$ defined in the hint is one-to-one. Suppose $X \neq Y$ and suppose $x \in X - Y$. If $f(a) = x$ where $a \in B$, when $a \in f_{-1}(X) - f_{-1}(Y)$, so $g(X) \neq g(Y)$.

$\square$




Exercise 3.10.

Solution. By the pairing function $\Gamma$ in Theorem 3.5 we have that $\omega_\alpha \times \omega_\alpha$ is a projection of $\omega_\alpha$, and therefore $P(\omega_\alpha \times \omega_\alpha)$ is a projection of $P(\omega_\alpha)$. Let $f$ be the function in the hint (and if $R$ is not a well-order, let $f(R) := 0$). We shall show that $\ran(f) = \omega_{\alpha+1}$.

$\ran(f) \supseteq \omega_{\alpha+1}$: Let $\beta < \omega_{\alpha+1}$. Then $\vert \beta\vert \leq \aleph_\alpha$, so there exists a one-to-one function $f : \beta \to \omega_\alpha$. Define a well-ordering $R \subseteq \omega_\alpha \times \omega_\alpha$ by:
$$ \begin{align*}
(\gamma,\delta) \in R \iff (\exists \xi < \beta)(\exists \zeta < \xi)(f(\zeta) = \gamma \wedge f(\xi) = \delta)
\end{align*}$$ Then clearly $R$ is a well order of order type $\beta$.

$\ran(f) \subseteq \omega_{\alpha+1}$: Let $R$ be a well order and let $\beta = f(R)$. We may then define $g : \beta \to \omega_\alpha$ by having $g(\gamma) := \alpha_\gamma$, where $\alpha_\gamma$ is the $\gamma^\text{th}$ ordinal under the well order $R$. The well order of $R$ also implies that $g$ must be one-to-one, so $\vert \beta\vert \leq \vert \alpha\vert < \omega_{\alpha+1}$. Hence $\beta \in \omega_{\alpha+1}$.

$\square$




Exercise 3.11.

Solution. By Lemma 3.3, $\aleph_{\alpha+1} < 2^{\aleph_{\alpha+1}}$. By Exercise 3.10, $\omega_{\alpha+1}$is a projection of $P(\omega_\alpha)$, so by Exercise 3.9 we have that $\vert P(\omega_{\alpha+1})\vert \leq \vert P(P(\omega_\alpha))\vert $, i.e. $2^{\aleph_{\alpha+1}} \leq 2^{2^{\aleph_\alpha}}$. Thus $\aleph_{\alpha+1} < 2^{2^{\aleph_\alpha}}$.

$\square$




Exercise 3.12.

Solution. There is a typo in the second statement: $\omega_\alpha$ is not the limit of $\c{\omega_\xi : \xi < \cf{\alpha}}$. Regardless, it's true that $\cf{\omega_\alpha} = \cf{\alpha}$. This follows from Lemma 3.7(ii) and $\omega_\alpha := \lim_{\xi\to\alpha} \omega_\xi$.

$\square$




Exercise 3.13.

Solution. We expand on the hint. Note that the mapping $f : \omega \times \alpha \to \omega_2$ is defined as:
$$ \begin{align*}
f(n,\beta) = \text{the $\beta^\text{th}$ element of $S_n$}
\end{align*}$$ where if $\beta \geq \alpha_n$ then $f(n,\beta) := 0$. Now $\vert \omega \times \alpha\vert = \aleph_0 \cdot \vert \alpha\vert \leq \omega_1$, so $\alpha \times \omega$ is a projection of $\omega_1$. Composing these two maps implies that $\omega_2$ is a projection of $\omega_1$, a contradiction.

$\square$




Exercise 3.14.

Solution. If $S$ is D-infinite, let $f,X$ be as in the hint. We shall show that $i \neq j$ implies $x_i \neq x_j$ (so $\lbrace x_n : n < <\omega\rbrace$ is indeed a countably infinite subset). Suppose $x_i = x_j$ for some $i,j < \omega$. WLOG suppose $i \leq j$, and write $j = i + k$. Then $f^{(i+k)}(x_0) = f^{(i)}(x_0)$. Since $f$ is one-to-one, $x_0 = f^{(k)}(x_0)$. Since $f^{(k)}(x_0) \in S - X$ but $\ran(f) \subseteq X$, we must have $k = 0$. Thus $i = j$.

Conversely, suppose $S$ has a countable subset $X \subseteq S$. Let $f : X \to \omega$ be one-to-one and onto. We define $g : S \to S$ by:
$$ \begin{align*}
g(x) :=
\begin{cases}
x, &\text{if $x \in S - X$} \\
f^{-1}(f(x + 1)), &\text{if $x \in X$}
\end{cases}
\end{align*}$$ Clearly $g$ is one-to-one. Furthermore, $\ran(g)$ is a proper subset of $S$ as $f^{-1}(0) \notin \ran(g)$. Thus $S$ is D-infinite.

$\square$




Exercise 3.15.


Exercise 3.15(i).

Solution. Suppose $A \cup B$ is D-infinite, so by Exercise 3.14 it contains a countable subset. Let $X \subseteq A \cup B$ be countable. Then $X = (X \cap A) \cup (X \cap B)$. By Exercise 3.2(i), $X \cap A$ and $X \cap B$ are at most countable. Since $A$ and $B$ are $D$-finite, they do not contain a countable subset. Thus, $X \cap A$ and $X \cap B$ are both finite. By Exercise 3.1(ii), $X$ is finite, a contradiction.

Suppose $A \times B$ is D-infinite with $X \subseteq A \times B$ countable. Let:
$$ \begin{gather*}
X_A := \lbrace a \in A : (\exists b \in B) \, (a,b) \in X\rbrace \\
X_B := \lbrace b \in B : (\exists a \in A) \, (a,b) \in X\rbrace
\end{gather*}$$ $X_A$ and $X_B$ are images under the mappings $(a,b) \mapsto a$ and $(a,b) \mapsto b$ of $X$ respectively, so by Exercise 3.2(iii) they are both finite. But observe that $X \subseteq X_A \times X_B$, so by Lemma 3.15(i).A and Exercise 3.1(i) $X$ is finite, a contradiction.

$\square$




Exercise 3.15(ii).

Solution. Let $A$ be a D-finite set and let $A'$ denote the set of all one-to-one sequences of elements of $A'$. Suppose $A'$ is D-infinite with $X \subseteq A'$ countable. Write $X = \lbrace s_i : i < \omega\rbrace$. Define $x_i \in A$ inductively as follows:
  1. $x_0 := s_0(0)$.
  2. $x_{n+1} := s_i(k)$, where $i$ is the least $i < \omega$ such that $\ran(s_i) \not\subseteq \lbrace x_j : j < i\rbrace$, and $k$ is the least $k < \length(s_i)$ such that $s_i(k) \notin \lbrace x_j : j < i\rbrace$.
If $x_n$ is well-defined for all $n$, then it forms a countable subset of $A$, a contradiction. To see that it is indeed well-defined, suppose not, so there exists a finite set $F$ and an $n$ such that for all $m \geq n$, $\ran(s_m) \subseteq F$. This implies that:
$$ \begin{align*}
\bigcup_{m < \omega} \ran(s_m) \subseteq \underbrace{F \cup \bigcup_{m < n} \ran(s_m)}_{\text{finite}}
\end{align*}$$ Therefore $X \subseteq [F']^{<\omega}$ for some finite set $F'$. Thus, if $\vert F'\vert = N$, then since all sequences in $X$ are one-to-one, we have:
$$ \begin{align*}
X \subseteq \bigcup_{i \leq N} \prod_{j < i} F'
\end{align*}$$ but RHS is finite by Lemma 3.15(i).A and Exercise 3.1(ii), a contradiction.

$\square$




Exercise 3.15(iii).

Solution. Let $I$ be D-finite, and for each $i \in I$ let $A_i$ be a D-finite set such that $i \neq j \implies A_i \cap A_j = \emptyset$. Let $A := \bigsqcup_{i \in I} A_i$. Suppose $A$ is D-infinite with a countable subset $X \subseteq A$. Write $X = \lbrace x_i : i < \omega\rbrace$.

Define $Y = \lbrace y_i : i < \omega\rbrace \subseteq X$ inductively as follows:
  1. $y_0 := x_0$. Let $i_0 \in I$ be the unique element of $I$ such that $y_0 \in A_{i_0}$. This $i_0$ is well-defined, as the set $\lbrace i \in I : x_0 \in i\rbrace$ is singleton.
  2. Let $y_n \in X$ be such that $y_n \notin \bigcup_{k < n} A_{i_k}$. Such a $y_n$ exists, for otherwise we have that $X \subseteq \bigcup_{k < n} A_{i_k}$. RHS is a finite union of D-finite set, which is D-finite by Exercise 3.15(i), a contradiction. Let $i_n \in I$ be (unique) such that $y_n \in A_{i_n}$.
In particular, this gives us $I' := \lbrace i_k : k < \omega\rbrace \subseteq I$, contradicting that $I$ is D-finite.

$\square$




Exercise 3.16.

Solution. As in the hint, it suffices to show that $\lbrace X \subseteq A : \vert X\vert = n\rbrace$ is non-empty for each $n$, in which it's clear that they are pairwise unequal, so $P(P(A))$ is D-infinite.

We induct on $n$ that $S_n := \lbrace X \subseteq A : \vert X\vert = n\rbrace$ is non-empty. $S_0$ is non-empty as $\emptyset \in S_0$. If $S_n$ is non-empty, let $X \in S_n$. Since $X \subseteq A$ is finite and $A$ is infinite, we have $A - X \neq \emptyset$, so let $x \in A - X$. Then $\vert X \cup \lbrace x\rbrace\vert = n + 1$ by Lemma 1.13.A, so $X \cup \lbrace x\rbrace \in S_{n+1}$, i.e. $S_{n+1}$ is non-empty.

$\square$